LSAT and Law School Admissions Forum

Get expert LSAT preparation and law school admissions advice from PowerScore Test Preparation.

 LSATer
  • Posts: 47
  • Joined: Nov 13, 2016
|
#33902
Hello!

So I was able to easily narrrow down the answer choices to A and C.

Cost of training high and require memorization--->~Buy
Buy-->~cost of training high or ~require memorization

Conclusion: successful --->~require memorization

When I put the two together :
Buy--->~require memorization
Successful---->~require memorization

Assumption : Buy---->Succesful

So I chose A but the answer is C. Where did I go wrong here?

Thank you in advance
LSATer
 Steven Palmer
PowerScore Staff
  • PowerScore Staff
  • Posts: 35
  • Joined: Feb 21, 2017
|
#33910
Hi LSATer,

You were definitely on the right track with this question! As with many justify questions, there is a missing link in this stimulus, between the premises and the conclusion. As you pointed out, that is between "successful" and "buying software". The stimulus tells us everything that is important about why a company will buy computer software, and then essentially concludes by saying, "To be successful, computer companies should do those things".

To make the conclusion follow logically, or in other words, become absolutely and provably correct, we need to say that companies buying software guarantees success. Choice (A) is not correct because it does not guarantee that buying = success. On the other hand, Choice (C) says that people purchasing the software is necessary to it being successful. It closes up that gap.

This is certainly a very challenging question, which is why it is found so near to the end of this LR section.

Hope this helps!
Steven
 LSATer
  • Posts: 47
  • Joined: Nov 13, 2016
|
#33951
Thank you for your response! I still don't full understand why A doesn't guarantee that the product will be successful if purchasers buy it.

Answer choice A: If most purchasers buy---> Product will be successful
Answer choice C: Successful---> prime purchasers buy

I don't quite understand why C is better than A. Is it the word "most" that doesn't make the purchasers buying sufficient for the product to be successful? Is this not a true conditional because of the word "most?"

Thanks again,

LSATer
 Robert Carroll
PowerScore Staff
  • PowerScore Staff
  • Posts: 1787
  • Joined: Dec 06, 2013
|
#33962
LSATer,

Let's make explicit every premise and the conclusion:

Premise: training costs high :arrow: prime purchasers buy it

Premise: demands memorization :arrow: training costs high

Conclusion: successful :arrow: demands memorization

"successful" is not connected to anything. In order to prephrase an appropriate connection, we have to see where the argument fails.

The chain of the premises is as follows:

demands memorization :arrow: training costs high :arrow: prime purchasers buy it

The contrapositive is then:

prime purchasers buy it :arrow: training costs high :arrow: demands memorization

That "demands memorization" is the necessary condition of the conclusion, so if "successful" were the sufficient condition of anything in this chain, the conclusion would follow. Thus:

successful :arrow: prime purchasers buy it

would allow the full chain to be as follows:

successful :arrow: prime purchasers buy it :arrow: training costs high :arrow: demands memorization

This now allows the conclusion to be drawn as the first sufficient condition in the chain and the last necessary condition in the chain.

Answer choice (C) matches that, once the Unless Equation is applied to the answer. Answer choice (A) is in the area of a Mistaken Reversal.

Robert Carroll
 Sophia123
  • Posts: 43
  • Joined: Mar 20, 2017
|
#34004
Hi!

The explanation above makes sense, but I am just confused on the diagramming of the second premise. I originally diagrammed it as costs high --> demands memorization of unfamiliar keys. I think I originally did it this way instead of the opposite because I saw the word "demand" and considered it to be something like "requires" which generally indicates the necessary condition. I don't fully understand why this premise is diagrammed in this way, and I would also like to know if there is any way to avoid the mistake that I made while diagramming in the future?

Thank you in advance!

-Sophia
 Kristina Moen
PowerScore Staff
  • PowerScore Staff
  • Posts: 230
  • Joined: Nov 17, 2016
|
#34022
Hi Sophia,

Let's spend a little more time on this sentence: "It is expensive to teach people a software package that demands the memorization of unfamiliar commands." The clause "that demands the memorization of unfamiliar commands" is simply describing the software package. You might also read it as: "It is expensive to teach people this type of software." Thus, if you are teaching someone that type of software, then it's expensive. It's not that ALL expensive software is of that type. So you would diagram it is as Software that demands memorization :arrow: training costs high.

It's as if I said "It's expensive to buy clothes that are made by Gucci." What does that mean? If the clothes are Gucci, then it's expensive. Gucci :arrow: expensive. It does not mean that all expensive clothes are Gucci.

However, you are right to see the word "demands" and think "requires." If the word "demands" was used as the main verb (rather than in a clause that describes another noun), then it could indicate a necessary condition. For example, if I said "Poodles demand regular brushing" then poodles :arrow: regular brushing. However, if I were describing what someone said i.e. "Harold demanded I turn off the TV," then it would not be a conditional reasoning indicator. Some English words have multiple meanings. I'm glad you're thinking beyond the list of Conditional Reasoning Indicators.

Hope that helps.
 LSATer
  • Posts: 47
  • Joined: Nov 13, 2016
|
#34241
As always, taking a break from the question and then coming back to it, helps me to see it much clearer. I see my error when writing my conditional statements. I combined "cost of training and require memorization" as sufficient for "~Buy"

Cost of training high and require memorization--->~Buy
Buy-->~cost of training high or ~require memorization

Looking at @Robert Caroll's explanation, the proper way to do it seems to be to break it up into the two premises. I read the stimuli incorrectly and put the two together. I am not sure if it makes a difference in this specific situation because when combined, I got to the same conclusion.

Conclusion: successful --->~require memorization

When I put the two together :
Buy--->~require memorization
Successful---->~require memorization

When I got to this point, I didn't know which should be the sufficient indicator (whether "buy" or "successful") to justify the conclusion. Thank you for clearing this up for me! The sufficient condition of anything in this chain based on the conclusion is "successful." Therefore Successful----->Prime Purchasers Buy

Thank you!

LSATer
 Kristina Moen
PowerScore Staff
  • PowerScore Staff
  • Posts: 230
  • Joined: Nov 17, 2016
|
#34342
Hi LSATer,

Nice job. Always look for the rogue element. Here, it's "successful" and "prime purchasers of computer software will not buy a software package." The first two premises gives us the inference that "prime purchasers of computer software will not buy a software package that demands the memorization of unfamiliar commands." The conclusion tells us that "successful :arrow: commercial computer software cannot require users to memorize unfamiliar commands." Of course, the contrapositive of that is: "commercial computer software requires users to memorize unfamiliar commands :arrow: not successful."

How can we get there?

P: software package demands the memorization of unfamiliar commands :arrow: prime purchases won't buy it
MISSING LINK: prime purchasers won't buy it :arrow: software package is not successful.
C: commercial computer software requires users to memorize unfamiliar commands :arrow: not successful.

You are absolutely correct that you need to link the two elements. Answer choice (C) gives us the missing link.

Get the most out of your LSAT Prep Plus subscription.

Analyze and track your performance with our Testing and Analytics Package.